Diễn Đàn MathScopeDiễn Đàn MathScope
  Diễn Đàn MathScope
Ghi Danh Hỏi/Ðáp Thành Viên Social Groups Lịch Ðánh Dấu Ðã Ðọc

Go Back   Diễn Đàn MathScope > Sơ Cấp > Đại Số và Lượng Giác > Các Bài Toán Đã Được Giải

News & Announcements

Ngoài một số quy định đã được nêu trong phần Quy định của Ghi Danh , mọi người tranh thủ bỏ ra 5 phút để đọc thêm một số Quy định sau để khỏi bị treo nick ở MathScope nhé !

* Nội quy MathScope.Org

* Một số quy định chung !

* Quy định về việc viết bài trong diễn đàn MathScope

* Nếu bạn muốn gia nhập đội ngũ BQT thì vui lòng tham gia tại đây

* Những câu hỏi thường gặp

* Về việc viết bài trong Box Đại học và Sau đại học


Trả lời Gởi Ðề Tài Mới
 
Ðiều Chỉnh Xếp Bài
Old 29-12-2010, 06:31 PM   #1
khtoan
+Thành Viên+
 
Tham gia ngày: Jan 2010
Đến từ: Đà Nẵng
Bài gởi: 155
Thanks: 23
Thanked 128 Times in 68 Posts
Một bất đẳng thức

Cho a,b,c>0, CMR
$2(a^{2}+1)(b^{2}+1)(c^{2}+1)\geq (a+1)(b+1)(c+1)(abc+1) $

[RIGHT][I][B]Nguồn: MathScope.ORG[/B][/I][/RIGHT]
 
khtoan is offline   Trả Lời Với Trích Dẫn
The Following User Says Thank You to khtoan For This Useful Post:
view (30-12-2010)
Old 29-12-2010, 06:37 PM   #2
daylight
+Thành Viên+
 
daylight's Avatar
 
Tham gia ngày: Dec 2009
Đến từ: Ha Noi
Bài gởi: 551
Thanks: 877
Thanked 325 Times in 188 Posts
Trích:
Nguyên văn bởi khtoan View Post
Cho a,b,c>0, CMR
$2(a^{2}+1)(b^{2}+1)(c^{2}+1)\geq (a+1)(b+1)(c+1)(abc+1) $
Sử dụng
$2(a^2+1)^3 \ge (a+1)^3(a^3+1) $
thật vậy vì nó tương đương $(a-1)^4(a^2+a+1) \ge 0 $
Lập các BDT tương tự rồi nhân lại sau đó sử dụng



Ta có điều phải chứng minh.

Thử sức bài tương tự :
$a,b,c,d>0 $ thỏa mãn$ \frac{1}{a}+\frac{1}{b}+\frac{1}{c}+\frac{1}{d}=4 $.Chứng minh

$\sum^{a,b,c,d}_{cyc} \sqrt[3]{\frac{a^3+b^3}{2}} +4 \le 2(a+b+c+d) $
[RIGHT][I][B]Nguồn: MathScope.ORG[/B][/I][/RIGHT]
 
daylight is offline   Trả Lời Với Trích Dẫn
The Following 2 Users Say Thank You to daylight For This Useful Post:
khtoan (29-12-2010), phantiendat_hv (30-12-2010)
Old 29-12-2010, 10:41 PM   #3
khtoan
+Thành Viên+
 
Tham gia ngày: Jan 2010
Đến từ: Đà Nẵng
Bài gởi: 155
Thanks: 23
Thanked 128 Times in 68 Posts
BĐT viết lại dưới dạng thuần nhất:
$\sum \sqrt[3]{\frac{a^{3}+b^{3}}{2}}+\frac{16abcd}{abc+bcd+acd+ abd}\leq 2(a+b+c+d) $
Sử dụng:
$\frac{a^{2}+b^{2}}{a+b}\geq \sqrt[3]{\frac{a^{3}+b^{3}}{2}} $
Điều đó tương đương với :
$2(a+b)\geq \frac{4ab}{a+b}+2\sqrt[3]{\frac{a^{3}+b^{3}}{2}} $
Làm 3 bất đẳng thức tương tự, cộng vế theo vế,ta chỉ cần chứng minh:
$\frac{ab}{a+b}+\frac{bc}{b+c}+\frac{cd}{c+d}+\frac {da}{d+a}\geq \frac{8abcd}{abc+bcd+acd+abd} $
$
\Leftrightarrow \sum \frac{1}{(a+b)cd}\geq \frac{8}{\sum abc} $ (hiển nhiên đúng theo cauchy schwarz)
Vậy bất đẳng thức đã đc chứng minh.
[RIGHT][I][B]Nguồn: MathScope.ORG[/B][/I][/RIGHT]
 

thay đổi nội dung bởi: khtoan, 29-12-2010 lúc 10:44 PM
khtoan is offline   Trả Lời Với Trích Dẫn
The Following 2 Users Say Thank You to khtoan For This Useful Post:
nhox12764 (30-12-2010), phantiendat_hv (30-12-2010)
Old 30-12-2010, 01:52 PM   #4
view
+Thành Viên+
 
view's Avatar
 
Tham gia ngày: Sep 2010
Đến từ: Nghe An
Bài gởi: 7
Thanks: 7
Thanked 2 Times in 1 Post
Bài bđt này trong đề thi vào chuyên toán của Nghệ An, loại này cũng quen với nhiều bác rùi nhưng mình cũng post lên nhé
cho a,b,c không âm thoả mãn a+b+c=3. Tìm max của
$P=a\sqrt{b}+b\sqrt{c}+c\sqrt{a}-\sqrt{abc}
$
[RIGHT][I][B]Nguồn: MathScope.ORG[/B][/I][/RIGHT]
 
__________________
Ta đang tìm mình giữa các vì sao
view is offline   Trả Lời Với Trích Dẫn
Old 31-12-2010, 12:13 PM   #5
daylight
+Thành Viên+
 
daylight's Avatar
 
Tham gia ngày: Dec 2009
Đến từ: Ha Noi
Bài gởi: 551
Thanks: 877
Thanked 325 Times in 188 Posts
Trích:
Nguyên văn bởi view View Post
Bài bđt này trong đề thi vào chuyên toán của Nghệ An, loại này cũng quen với nhiều bác rùi nhưng mình cũng post lên nhé
cho a,b,c không âm thoả mãn a+b+c=3. Tìm max của
$P=a\sqrt{b}+b\sqrt{c}+c\sqrt{a}-\sqrt{abc}
$
dạng này mình cũng vài lần thấy ,giả sử $\sqrt{b} $ nằm giữa $\sqrt{a} $,và $\sqrt{c} $

rồi sử dụng $\sqrt{c}(\sqrt{c}-\sqrt{b})(\sqrt{b}-\sqrt{a}) \ge 0 $

ta được

$P \le P-\sqrt{c}(\sqrt{c}-\sqrt{b})(\sqrt{b}-\sqrt{a})=\sqrt{b}(a
+c)=\sqrt{b}(3-b) $

đến đây thì tìm max dễ

[RIGHT][I][B]Nguồn: MathScope.ORG[/B][/I][/RIGHT]
 
daylight is offline   Trả Lời Với Trích Dẫn
The Following User Says Thank You to daylight For This Useful Post:
view (31-12-2010)
Old 31-12-2010, 04:41 PM   #6
winwave
+Thành Viên+
 
winwave's Avatar
 
Tham gia ngày: Jul 2010
Đến từ: Đà Nẵng
Bài gởi: 87
Thanks: 23
Thanked 40 Times in 24 Posts
Gửi tin nhắn qua Yahoo chát tới winwave
Trích:

[RIGHT][I][B]Nguồn: MathScope.ORG[/B][/I][/RIGHT]
 
winwave is offline   Trả Lời Với Trích Dẫn
The Following 2 Users Say Thank You to winwave For This Useful Post:
daylight (31-12-2010), view (31-12-2010)
Old 13-09-2011, 12:03 AM   #7
xtungftu
+Thành Viên+
 
Tham gia ngày: Aug 2011
Đến từ: Hiện ở HN
Bài gởi: 41
Thanks: 20
Thanked 14 Times in 11 Posts
Gửi tin nhắn qua Yahoo chát tới xtungftu
Trích:
Nguyên văn bởi daylight View Post
dạng này mình cũng vài lần thấy ,giả sử $\sqrt{b} $ nằm giữa $\sqrt{a} $,và $\sqrt{c} $

rồi sử dụng $\sqrt{c}(\sqrt{c}-\sqrt{b})(\sqrt{b}-\sqrt{a}) \ge 0 $

ta được

$P \le P-\sqrt{c}(\sqrt{c}-\sqrt{b})(\sqrt{b}-\sqrt{a})=\sqrt{b}(a
+c)=\sqrt{b}(3-b) $

đến đây thì tìm max dễ

[RIGHT][I][B]Nguồn: MathScope.ORG[/B][/I][/RIGHT]
 

thay đổi nội dung bởi: xtungftu, 13-09-2011 lúc 01:15 PM Lý do: Không sử dụng ngôn ngữ chat
xtungftu is offline   Trả Lời Với Trích Dẫn
Old 13-09-2011, 07:40 AM   #8
shinomoriaoshi
+Thành Viên+
 
Tham gia ngày: Mar 2010
Đến từ: Tuy Hòa
Bài gởi: 198
Thanks: 198
Thanked 129 Times in 72 Posts
Trích:
Nguyên văn bởi xtungftu View Post
Ở đây ko có vai trò bình đẳng của $ a,b,c $ nên ko thể giả sử như vậy
Theo mình nghĩ là được, vì chỉ cần a, b, c có vai trò hoán vị vòng quanh cho nhau, không cần vai trò bình đẳng.
[RIGHT][I][B]Nguồn: MathScope.ORG[/B][/I][/RIGHT]
 
shinomoriaoshi is offline   Trả Lời Với Trích Dẫn
Old 13-09-2011, 10:41 AM   #9
xtungftu
+Thành Viên+
 
Tham gia ngày: Aug 2011
Đến từ: Hiện ở HN
Bài gởi: 41
Thanks: 20
Thanked 14 Times in 11 Posts
Gửi tin nhắn qua Yahoo chát tới xtungftu
Trích:
Nguyên văn bởi shinomoriaoshi View Post
Theo mình nghĩ là được, vì chỉ cần a, b, c có vai trò hoán vị vòng quanh cho nhau, không cần vai trò bình đẳng.

[RIGHT][I][B]Nguồn: MathScope.ORG[/B][/I][/RIGHT]
 

thay đổi nội dung bởi: xtungftu, 13-09-2011 lúc 01:16 PM Lý do: Không sử dụng ngôn ngữ chat
xtungftu is offline   Trả Lời Với Trích Dẫn
Old 22-09-2011, 09:36 AM   #10
bboy114crew
+Thành Viên+
 
Tham gia ngày: Oct 2010
Đến từ: Dòng thời gian...
Bài gởi: 294
Thanks: 290
Thanked 189 Times in 91 Posts
Gửi tin nhắn qua Yahoo chát tới bboy114crew
Trích:
Nguyên văn bởi khtoan View Post
Cho a,b,c>0, CMR
$2(a^{2}+1)(b^{2}+1)(c^{2}+1)\geq (a+1)(b+1)(c+1)(abc+1) $
Trích:
Nguyên văn bởi daylight View Post
Sử dụng
$2(a^2+1)^3 \ge (a+1)^3(a^3+1) $
thật vậy vì nó tương đương $(a-1)^4(a^2+a+1) \ge 0 $
Lập các BDT tương tự rồi nhân lại sau đó sử dụng



Ta có điều phải chứng minh.

Thử sức bài tương tự :
$a,b,c,d>0 $ thỏa mãn$ \frac{1}{a}+\frac{1}{b}+\frac{1}{c}+\frac{1}{d}=4 $.Chứng minh

$\sum^{a,b,c,d}_{cyc} \sqrt[3]{\frac{a^3+b^3}{2}} +4 \le 2(a+b+c+d) $
Cách 2:
đặt :$a=\frac{1-x}{1+x};b=\frac{1-y}{1+y};c=\frac{1-z}{1+z} $
khi đó:
$\frac{a^2+1}{a+1}=\frac{x^2+1}{x+1}; $
$\frac{b^2+1}{b+1}=\frac{y^2+1}{y+1} $
$\frac{c^2+1}{c+1}=\frac{z^2+1}{z+1} $
$abc+1=\frac{2(xy+yz+zx+1)}{(x+1)(y+1)(z+1)} $
BDT trở thành:
$(x^2+1)(y^2+1)(z^2+1) \geq xy+yz+zx+1 $
$\Leftrightarrow \sum x^2y^2+ \sum x^2 \geq \sum xy $
theo AM-GM:
$\sum x^2 = \sum \frac{x^2+y^2}{2} \geq \sum|xy| \geq \sum xy $
và $\sum x^2y^2 \geq 0 $
$\Rightarrow dpcm $
dấu = xảy ra khi a=b=c=1
[RIGHT][I][B]Nguồn: MathScope.ORG[/B][/I][/RIGHT]
 
__________________
Thay đổi tất cả và mãi mãi......
Offline...
bboy114crew is offline   Trả Lời Với Trích Dẫn
Trả lời Gởi Ðề Tài Mới

Bookmarks

Ðiều Chỉnh
Xếp Bài

Quuyền Hạn Của Bạn
You may not post new threads
You may not post replies
You may not post attachments
You may not edit your posts

BB code is Mở
Smilies đang Mở
[IMG] đang Mở
HTML đang Tắt

Chuyển đến


Múi giờ GMT. Hiện tại là 12:57 PM.


Powered by: vBulletin Copyright ©2000-2024, Jelsoft Enterprises Ltd.
Inactive Reminders By mathscope.org
[page compression: 81.73 k/93.10 k (12.22%)]